Difference between revisions of "2017 AMC 8 Problems/Problem 4"

m (Solution:)
Line 6: Line 6:
 
==Solution==
 
==Solution==
 
We can approximate <math>7,928,564</math> to <math>8,000,000,</math> and <math>0.000315</math> to <math>0.0003.</math> Multiplying the two yields <math>2400.</math> This gives our answer to be <math>\boxed{\textbf{(D)}\ 2400}.</math>
 
We can approximate <math>7,928,564</math> to <math>8,000,000,</math> and <math>0.000315</math> to <math>0.0003.</math> Multiplying the two yields <math>2400.</math> This gives our answer to be <math>\boxed{\textbf{(D)}\ 2400}.</math>
 +
 +
==See Also==
 +
{{AMC8 box|year=2017|num-b=3|num-a=5}}
 +
 +
{{MAA Notice}}

Revision as of 15:24, 22 November 2017

Problem 4

When $0.000315$ is multiplied by $7,928,564$ the product is closest to which of the following?

$\textbf{(A) }210\qquad\textbf{(B) }240\qquad\textbf{(C) }2100\qquad\textbf{(D) }2400\qquad\textbf{(E) }24000$

Solution

We can approximate $7,928,564$ to $8,000,000,$ and $0.000315$ to $0.0003.$ Multiplying the two yields $2400.$ This gives our answer to be $\boxed{\textbf{(D)}\ 2400}.$

See Also

2017 AMC 8 (ProblemsAnswer KeyResources)
Preceded by
Problem 3
Followed by
Problem 5
1 2 3 4 5 6 7 8 9 10 11 12 13 14 15 16 17 18 19 20 21 22 23 24 25
All AJHSME/AMC 8 Problems and Solutions

The problems on this page are copyrighted by the Mathematical Association of America's American Mathematics Competitions. AMC logo.png